Question

Based on the CAPM, a stock has a required return of 16%. The beta of the...

Based on the CAPM, a stock has a required return of 16%. The beta of the stock is 2.5 and the risk-free rate is 4%. What is the market risk premium? 12% 4.4% 8.8% 6.30% None of the above

0 0
Add a comment Improve this question Transcribed image text
Answer #1

required return= risk-free rate+Beta*market risk premium

16=4+2.5*market risk premium

market risk premium=(16-4)/2.5

=4.8%

Hence the correct option is None of the above,

Add a comment
Know the answer?
Add Answer to:
Based on the CAPM, a stock has a required return of 16%. The beta of the...
Your Answer:

Post as a guest

Your Name:

What's your source?

Earn Coins

Coins can be redeemed for fabulous gifts.

Not the answer you're looking for? Ask your own homework help question. Our experts will answer your question WITHIN MINUTES for Free.
Similar Homework Help Questions
ADVERTISEMENT
Free Homework Help App
Download From Google Play
Scan Your Homework
to Get Instant Free Answers
Need Online Homework Help?
Ask a Question
Get Answers For Free
Most questions answered within 3 hours.
ADVERTISEMENT
ADVERTISEMENT
ADVERTISEMENT